RegistrierenRegistrieren   LoginLogin   FAQFAQ    SuchenSuchen   
Rohrfüllhöhe in Abhängigkeit vom Volumenstrom
 
Neue Frage »
Antworten »
    Foren-Übersicht -> Mechanik
Autor Nachricht
tectone



Anmeldungsdatum: 01.08.2020
Beiträge: 13

Beitrag tectone Verfasst am: 17. Nov 2020 19:33    Titel: Rohrfüllhöhe in Abhängigkeit vom Volumenstrom Antworten mit Zitat

Hey Leute,

ich habe folgende Frage zur Strömungslehre:
Gibt es irgendeine Möglichkeit zu ermitteln, wie voll ein horizontales Rohr (Welche Höhe h hat der Querschnitt des strömenden Wasservolumens) mit Innendurchmesser D ist ohne Steigungswinkel, wenn ein bestimmter variabler Volumenstrom Q an einem Ende angelegt wird?

Es müsste ja gelten h ~ Q bei d=const. und h~ (1/d) bei Q=const.

Gibt es dafür einen einfachen mathematischen/physikalischen Ansatz wie man h errechnen kann oder muss man das aufwendig per Strömungssimulation analyisieren (Solidworks / Ansys)? Es wäre gut zu wissen, welcher Volumenstrom ohne nennenswerten Druckverlust notwendig ist um h=d zu erreichen.

Zum Hintergrund: Normalerweise stehen Rohrleitungen ja unter Druck, da Ventile den Fluss regeln und den Rohrquerschnitt verringern und so den Fluss verringern. Folglich sind die Rohe immer voll gefüllt. Ich interessiere mich aber für den Grenzfall, wenn der anliegende Gegendruck nur durch den Rohrquerschnitt und die Trägheit des Fördermediums hervorgerufen wird, also sehr gering ist. Folglich sollten vor allem die Viskosität und die Dichte eine Rolle darüber spielen, welcher Volumenstrom erforderlich ist (gehen ja in die Berechnung des Druckabfalls mit ein), um das Rohr voll auszufüllen. Würde man das Experiment mit Pudding oder Quecksilber durchführen, würde meiner Vorstellung nach der Gegendruck bei gleichem Rohrdurchmesser größer werden, und es wäre ein geringerer Volumenstrom notwendig um völlige Füllung zu erreichen, da bereits vorhandenes Fluid im Rohr quasi nachströmendes "blockiert" und das Nachströmen weiter "erschwert". Bis jetzt habe ich noch keine Beschreibung dessen gefunden. Aber vlt. übersehe ich da einfach nur die entscheidende Gleichung! Wenn ihr mir da weiterhelfen könntet wäre das super!


Zuletzt bearbeitet von tectone am 19. Nov 2020 22:40, insgesamt einmal bearbeitet
Frankx



Anmeldungsdatum: 04.03.2015
Beiträge: 982

Beitrag Frankx Verfasst am: 18. Nov 2020 08:49    Titel: Antworten mit Zitat

Zitat:
Gibt es irgendeine Möglichkeit zu ermitteln, wie voll ein horizontales Rohr (Welche Höhe h hat der Querschnitt des strömenden Wasservolumens) mit Innendurchmesser D ist ohne Steigungswinkel, wenn ein bestimmter variabler Volumenstrom Q an einem Ende angelegt wird?


Das dürfte daran scheitern, dass dann das Rohr über seine Länge nicht überall den gleichen Füllstand aufweist.
Es wird am Zulauf voller sein, als am Auslauf.

Für die Berechnung dieses Verlaufes spielen tatsächlich viele Parameter (Volumenstrom, Rohrlänge, Querschnitt, Viskosität, Strömungsverhältnisse (laminar/turbulent), möglicherweise Dichte usw. eine Rolle.

Man könnte das sicher in eine Formel fassen, aber es gibt auch Simulationsprogramme welche die Fluiddynamik berechnen können.



.
Mathefix



Anmeldungsdatum: 05.08.2015
Beiträge: 5863
Wohnort: jwd

Beitrag Mathefix Verfasst am: 19. Nov 2020 18:41    Titel: Antworten mit Zitat

Es handelt sich um ein Gerinne.
Da kein Gefälle besteht und Reibung unberücksichtigt bleiben soll, gilt



Es muss die Höhe h des Segments eines Kreises mit gegebener Fläche/Radius bestimmt werden.

Falls ich mich nicht verrechnet habe:

A_s = Fläche des Kreissegments
r = Radius
h = Höhe des Kreissegments







Darin beseht die eigentliche Schwierigkeit der Aufgabe.

Bei einem rechteckigen Querschnitt mit gegebener Breite ist es einfach.
Brillant



Anmeldungsdatum: 12.02.2013
Beiträge: 1973
Wohnort: Hessen

Beitrag Brillant Verfasst am: 19. Nov 2020 20:22    Titel: Antworten mit Zitat

Frankx hat Folgendes geschrieben:
Es wird am Zulauf voller sein, als am Auslauf.
Dafür muss sich wohl die Fließ-Geschwindigkeit erhöhen. Kann man das in einem waagerechten Rohr voraussetzen?

Experiment: Ich halte meine Zahnpasta-Tube horizontal, schraube die Kappe ab. Am Auslauf tut sich nichts, das „Rohr“ bleibt bis oben gefüllt. Wenn Zulauf wäre, würde sich die Füllung nicht ändern. Nur das, was zuläuft, verlässt auch das Rohr.

_________________
Glaubt nicht dem Hörensagen ... oder eingewurzelten Anschauungen, auch nicht den Worten eines verehrten Meisters; sondern was ihr selbst gründlich geprüft und als euch selbst und anderen zum Wohle dienend erkannt habt, das nehmt an. Siddhartha Gautama
Frankx



Anmeldungsdatum: 04.03.2015
Beiträge: 982

Beitrag Frankx Verfasst am: 19. Nov 2020 22:41    Titel: Antworten mit Zitat

Zitat:
Da kein Gefälle besteht und Reibung unberücksichtigt bleiben soll, gilt


Die Rohrreibung und die innere Reibung der Flüssigkeit spielen bei solchen Vorgängen eine wichtige Rolle und können meist nicht vernachlässigt werden.

Zitat:
Dafür muss sich wohl die Fließ-Geschwindigkeit erhöhen. Kann man das in einem waagerechten Rohr voraussetzen?


Ja, wenn das Rohr einen freien Auslauf hat. Und das setze ich mal voraus, da sonst die Aufgabe wenig Sinn ergibt.

Der Rohrwiderstand wächst mit der verbleibenden Rohrlänge.
D.h. er ist am (freien) Auslauf am geringsten. Damit steig der Wasserspiegel zum Zulauf hin stetig an.



.
tectone



Anmeldungsdatum: 01.08.2020
Beiträge: 13

Beitrag tectone Verfasst am: 19. Nov 2020 23:23    Titel: Antworten mit Zitat

Mathefix hat Folgendes geschrieben:
Es handelt sich um ein Gerinne.
Da kein Gefälle besteht und Reibung unberücksichtigt bleiben soll, gilt



Es muss die Höhe h des Segments eines Kreises mit gegebener Fläche/Radius bestimmt werden.

Falls ich mich nicht verrechnet habe:

A_s = Fläche des Kreissegments
r = Radius
h = Höhe des Kreissegments







Darin beseht die eigentliche Schwierigkeit der Aufgabe.

Bei einem rechteckigen Querschnitt mit gegebener Breite ist es einfach.


Ich glaube kaum, dass der Querschnitt hier einen Unterschied macht. Aus analytischer Sicht mag das so sein, wir reden hier nach meinem Verständnis nicht über ein klassisches analytisches Problem (man möge mich gerne berichtigen). Reibung kann hier auch nicht unberücksichtigt bleiben, da die innere Reibung der Wassermoleküle und Die Wechselwirkung mit der Rohrinnenwand den Volumenkörper beeinflussen wird. Ich dachte erst es gibt da eine Gleichung, konnte aber nach zwei Wochen bisher nicht wirklich was finden. Das was ich gefunden habe, bezieht sich in der Tat auf Flüsse, die durch ein Gefälle hervorgerufen werden und bei denen die Rohe nicht vollgefüllt sind. Eine ältere Software dazu findet man unter dem Namen "Pipe advisor". Das was Frankx zu Anfang gesagt hat macht schon Sinn.
Hier handelt es nach meiner weiteren Recherche um ein Problem für eine Computational Fluid Dynamics Simulation, genauer gesagt um eine transiente Berechnung mit einem mehrphasigem System zwei nicht mischbarer Phasen (Wasser, Luft) die vereinfacht als inkompressibel angenommen werden, wobei natürlich die Gravitation nicht vernachlässigt werden kann. Ich bin gerade dabei zu versuchen, das zu simulieren. Allerdings bin ich noch dabei, mich in die Thematik einzulesen. Falls jemand hier mehr Erfahrung hat, würde ich da gerne ein paar Fragen stellen.
tectone



Anmeldungsdatum: 01.08.2020
Beiträge: 13

Beitrag tectone Verfasst am: 19. Nov 2020 23:52    Titel: Antworten mit Zitat

Frankx hat Folgendes geschrieben:


Der Rohrwiderstand wächst mit der verbleibenden Rohrlänge.
D.h. er ist am (freien) Auslauf am geringsten. Damit steig der Wasserspiegel zum Zulauf hin stetig an.


Der Rohrwiderstand (der Widerstand der von der Rohrinnenseite ausgeht) ist so wie ich das sehe an jeder Stelle gleich. Warum: Die Rauigkeit und der Querschnitt sind ja unverändert. Allerdings ist beim Einlauf durch die Reibung die Masse an Fluid am größten, die einem freien Weiterströmen entgegenwirkt. Am Auslauf ist die Masse die einem Weiterströmen entgegenwirkt gleich null.
In Übereinstimmung mit Bernoulli muss bei einem kleinerem Wert für h und damit A natürlich V anwachsen, damit das Gleichgewicht im Zu- und Abfluss gewahrt bleibt. Der Fluss am Auslauf ist also entsprechend schneller wie der Pfropfen am Einlauf langsamer ist.
Ich glaube beim Wasser ist dieser Abfall an Höhe aufgrund der geringen Viskosität nur schwer im Alltagsgebrauch zu beobachten. Aber bei z. B. Honig würde das jedem sofort einleuchten.
Mathefix



Anmeldungsdatum: 05.08.2015
Beiträge: 5863
Wohnort: jwd

Beitrag Mathefix Verfasst am: 20. Nov 2020 09:02    Titel: Antworten mit Zitat

@tectone
Natürlich kann man Gefälle und die Reibung an den benetzten Flächen analytisch berücksichtigen.
Die Schwierigkeit der Aufgabe liegt darin, die Gleichung nach der Höhe h des Kreissegments umzustellen.
Brillant



Anmeldungsdatum: 12.02.2013
Beiträge: 1973
Wohnort: Hessen

Beitrag Brillant Verfasst am: 20. Nov 2020 09:55    Titel: Antworten mit Zitat

Frankx hat Folgendes geschrieben:
Damit steig der Wasserspiegel zum Zulauf hin stetig an.
Irgendwo wurde Viskosität erwähnt. Da habe ich übersehen, dass der Fragesteller Wasser meint.

Aber Wasser fließt nicht so leicht aus einem liegenden Gartenschlauch. Das merkt man, wenn man den Schlauch aufrollt, der ist voll mit Wasser.

Je kleiner der Durchmesser des Rohres / Schlauchs, desto mehr „haftet“ Wasser. Bei Kapillarröhrchen kommt es gar nicht freiwillig raus.

_________________
Glaubt nicht dem Hörensagen ... oder eingewurzelten Anschauungen, auch nicht den Worten eines verehrten Meisters; sondern was ihr selbst gründlich geprüft und als euch selbst und anderen zum Wohle dienend erkannt habt, das nehmt an. Siddhartha Gautama
Mathefix



Anmeldungsdatum: 05.08.2015
Beiträge: 5863
Wohnort: jwd

Beitrag Mathefix Verfasst am: 20. Nov 2020 10:15    Titel: Antworten mit Zitat

Zur Ermittlung der Reibungsverluste an den benetzten Flächen muss der hydraulische Durchmesser der benetzten Querschnitssfläche berechnet werden.
Dieser hängt von der Querschnittssfläche und dem Umfang ab, diese wiederum von h .

Für ein Kreissegment ist der hydraulische Durchmesser analytisch nicht bestimmbar.
Neue Frage »
Antworten »
    Foren-Übersicht -> Mechanik